LSAT RC Question for 175+ Scorers Forum

Prepare for the LSAT or discuss it with others in this forum.
Post Reply
PanjandrumOfReason

New
Posts: 31
Joined: Fri Aug 18, 2017 3:35 am

LSAT RC Question for 175+ Scorers

Post by PanjandrumOfReason » Fri Feb 22, 2019 1:04 am

Please, consider the following statement by an author of a passage and the following question.

Author: Opponents of my view argue that because all people have the potential to become evil, no one should be allowed to obtain a firearm.

Which of the following statements is the author of the passage most likely to agree with?
A. All people have the potential to become evil.

Based on this one statement from the passage alone, is A a correct answer to this question? (I understand that the question says "most likely," and thus, technically, one would need to see all the answer choices before being able to determine which is correct. But every test prep advice and actual LSAT question of this type that I've seen suggests that there will only be one answer choice that the author actually agrees with, so it would never be necessary to differentiate the degree to which the author agrees with multiple answer choices.)

Actually, there was an actual question on an undisclosed LSAT that I took that had basically a parallel situation to the question I pose here. An argument that I just read between two people on Reddit essentially reduces to whether or not A would be a correct answer to the above question, so it reminded me of that LSAT question, and that is why I am asking it here.

When I took that undisclosed LSAT, I did in fact end up choosing the parallel to my answer choice A here because the other four choices seemed wrong, but I did not feel good about it. I don't believe that it can be properly inferred from the author's statement above that the author agrees with answer choice A. The author is simply restating their opponent's argument without giving any indication as to whether they believe the premise or conclusion of that argument. I did score 170+ though, so who knows?!

What if I changed the wording of the author's statement as follows?
Author: Opponents of my view argue that all people have the potential to become evil, and therefore no one should be allowed to obtain a firearm.
Using this wording, I believe it becomes clear that A is not a correct answer choice.

And what about this wording?
Author: Opponents of my view argue that the fact that all people have the potential to become evil implies that no one should be allowed to obtain a firearm.
This one is probably the hairiest for me. But since its meaning is identical to my initial wording, I would still say A is not a correct answer here.

All comments appreciated.

AJordan

Silver
Posts: 533
Joined: Wed Oct 05, 2016 3:48 am

Re: LSAT RC Question for 175+ Scorers

Post by AJordan » Fri Feb 22, 2019 3:52 am

The wording doesn’t matter. They’re all the same, “transferred attribution” error that even weaker students learn to avoid pretty easily.

PanjandrumOfReason

New
Posts: 31
Joined: Fri Aug 18, 2017 3:35 am

Re: LSAT RC Question for 175+ Scorers

Post by PanjandrumOfReason » Fri Feb 22, 2019 9:19 am

I don't find it as clear-cut as you seem to. I could reasonably see an author using that third wording to indicate that they agree with the premise but not the conclusion of their opponents' argument. I see it as ambiguous language that lends itself to two interpretations.

For fun, let's add a couple more answer choices:
B. At least some people should be allowed to obtain a firearm. (Incorrect)
C. At least some people do not have the potential to become evil or at least some people should be allowed to obtain a firearm. (Correct?)

And what about a different question:
Which of the following statements are the opponents of the author most likely to agree with?
A. All people have the potential to become evil.

Is A an incorrect answer here because the author may be misrepresenting what their opponents are arguing? I think I might be overthinking things.

AJordan

Silver
Posts: 533
Joined: Wed Oct 05, 2016 3:48 am

Re: LSAT RC Question for 175+ Scorers

Post by AJordan » Fri Feb 22, 2019 10:31 am

I've been working on/teaching this test for going on three years and I haven't seen any answer like the (c) you posited. B is obviously incorrect. As to the crux of your argument, you can essentially swap the word 'that' in your stimulus with a comma and quotes. That's LSAT equivalent construction. So then it becomes,

Opponents of my view argue, "the fact that all people have the potential to become evil implies that no one should be allowed to obtain a firearm."

This is, admittedly, unclear colloquially but it's not unclear on the LSAT; this is equivalent construction. I'm open to be proven wrong but I'll need you to point me toward an actual question that follows this construction. And besides, you can't do RC questions in a vacuum like this. It opens the possibility of using the wrong piece of the passage to try and answer the question. I get where you're coming from with the "most likely to agree" bit part being a bit harrier than your standard LR MSS, but that's all the more reason to require the entire passage to even hypothesize.

The prep materials are correct, the test is designed so there is only one correct answer choice. Of the thousands of LR/RC questions I've done I have a real problem with fewer than ten of them, and none of them look like this.

Can you reference me an actual passage/question? I'll try and digest it further.

Seanathon

New
Posts: 12
Joined: Mon Jul 16, 2018 7:55 pm

Re: LSAT RC Question for 175+ Scorers

Post by Seanathon » Thu Feb 28, 2019 11:01 pm

Not a 175+ scorer but I hope I can help explain it in a more colloquial way. If the author's "opponents" are arguing that because all people have the potential to become evil, no one should be allowed to obtain a firearm, what would suggest that the author would agree that all people have the potential to become evil? In other words, the author's opponents are presumably of an opposing position to the author, so why would the author be at all likely to agree with something that would support the opponents' premise? I could be missing something, but I don't see anything in that statement that would support A being a correct answer.

Hope this helped! If I've made a mistake or made things worse let me know haha

Want to continue reading?

Register now to search topics and post comments!

Absolutely FREE!


Post Reply

Return to “LSAT Prep and Discussion Forum”